GMAT Stuff

You might also like

Download as docx, pdf, or txt
Download as docx, pdf, or txt
You are on page 1of 14

TIMING

You have 75 minutes to complete the Quantitative (Math) Section. Note: the clock on test day will not
show seconds until the final five minutes.
(QUANTITATIVE) MATH HELP
Data Sufficiency questions consist of a question and two statements labeled (1) and (2). You have to
decide whether the data given in the statements are SUFFICIENT for solving the question. Every Data
Sufficiency question has five choices: A) Statement (1) ALONE is sufficient, but statement (2) alone is
not sufficient to answer the question asked; B) Statement (2) ALONE is sufficient, but statement (1) alone
is not sufficient to answer the question asked; C) Statements (1) and (2) TAKEN TOGETHER are
sufficient to answer the question, even though NEITHER statement BY ITSELF is sufficient. D) Either
statement BY ITSELF is sufficient to answer the question. E) Statements (1) and (2) TAKEN
TOGETHER are NOT sufficient to answer the question, meaning that further information would be
needed to answer the question.

Question 2.

If n is odd, is x + 1 odd?
(1) x = n + 1
(2) n = 13

A. Statement (1) BY ITSELF is sufficient to answer the question, but statement (2) by itself is not.
B. Statement (2) BY ITSELF is sufficient to answer the question, but statement (1) by itself is not.
C. Statements (1) and (2) TAKEN TOGETHER are sufficient to answer the question, even though
NEITHER statement BY ITSELF is sufficient.
D. Either statement BY ITSELF is sufficient to answer the question.
E. Statements (1) and (2) TAKEN TOGETHER are NOT sufficient to answer the question, meaning that
further information would be needed to answer the question.

(A) Go through the statements. If n is odd and x = n + 1, then x is even. (An odd number + 1 is always an
even number, and vice versa). If x is even, then x + 1 is odd, thus statement 1 is sufficient.
Statement 2 tells you that n = 13. Since it doesn't tell you anything about x, however, statement 2 is
insufficient. Since statement 1 is sufficient, and statement 2 is not, the answer is A.

Question 3.

A fish tank is half full of water. When 10 liters are added, the tank is 7/8 full. What is the capacity of the
tank in liters?

A. 24 3/8
B. 26 2/3
C. 28 1/3
D. 30 3/8
E. 32 1/8
(B) Set L as the number of liters that the tank holds. Translate the question: half of the tank plus ten liters
equals 7/8 of the tank:
L/2 + 10 = (7/8)L.
This is rewritten as 10 = (7/8)L – (4/8)L = (3/8)L. (Subtract L/2 from both sides and convert L/2 into
(4/8)L).
(3/8)L = 10
Multiply both sides by 8/3
Therefore L = (8 × 10)/3 = 80/3 = 26 and 2/3 liters.

Question 4.

When Sarah received 10x coins, he then had 5y + 1 times as many coins as he had originally. In terms of x
and y, how many coins did Sarah have originally?

A. 10x (5y + 1)
B. (5y+1)/10x
C. 2x/y
D. 10/ (5y + 1)
E. None of the above

(C) Set s as the number of coins Sarah originally had. Thus, s + 10x = the number of coins Sarah had after
receiving 10x coins.
We are told:
s + 10x = (5y + 1)s
s + 10x = 5ys + s
10x = 5ys
s = 10x/5y
s = 2x/y

Question 5.

Oil costs $4 per liter and diesel costs $5.24 per liter. If the price of oil rises by 10% a month, and the price
of diesel is unchanged, how many months will it take until a liter of oil costs more than a liter of diesel?

A. 2
B. 3
C. 4
D. 5
E. 6

(B) The price of oil rises 10% (to $4.40) after one month. The next month it rises another 10% (multiply
by 1.1) to raise it to $4.84. At the end of the third month it rises to $5.32 ($4.84 × 1.1). This makes it
higher than diesel ($5.24).
Question 7.

If it takes 6 printing presses 4 hours to print 5000 newspapers, how long should it take 3 presses to print
3000 newspapers?

A. 3 hrs., 20 min.
B. 4 hrs., 20 min.
C. 4 hrs., 48 min.
D. 5 hrs., 48 min.
E. 6 hrs., 50 min.

(C) The first step in solving this problem is determining the rate of the printing presses. If it takes 6
presses 4 hours to print 5000 newspapers, the rate is 5000/4 newspapers/hour or 1250 per hour. 3 presses
are 1/2 as many presses, so we multiply the rate by 1250 by 1/2 or 625 per hour. Once we know the rate,
we can solve for time by using the equation time = amount/rate. The amount (number of papers) equals
3000 and the rate is 625 per hour, therefore, Time = 3000/625. Simplify that to Time = 3000/625, and
divide by 125 to simplify again to Time = 24/5 = 4 4/5 hours. 4 4/5 hours can be converted into minutes
by multiplying by 60, so the answer is 6 hours and 48 min.

Question 8.

A science class has a ratio of girls to boys of 4 to 3. If the class has a total of 35 students, how many more
girls are there than boys?

A. 20
B. 15
C. 7
D. 5
E. 1

(D) This is a problem that involves ratios. Your parts are made up of girls and boys at a ratio of 4:3. Your
total is 7. Since we know there are 35 students total and this number is broken into a ratio totaling 7,
divide 35 by 7 to get 5. Multiply 4 by 5 to get 20 girls and 3 by 5 to get 15 boys. The question asks for the
difference between the number of girls and the number of boys, so your answer is 5.

Question 10

How many sweaters does Jim own?


(1) Jim owns 12 wool sweaters
(2) 60% of the sweaters Jim owns are not wool.

A. Statement (1) BY ITSELF is sufficient to answer the question, but statement (2) by itself is not.
B. Statement (2) BY ITSELF is sufficient to answer the question, but statement (1) by itself is not.
C. Statements (1) and (2) TAKEN TOGETHER are sufficient to answer the question, even though
NEITHER statement BY ITSELF is sufficient.
D. Either statement BY ITSELF is sufficient to answer the question.
E. Statements (1) and (2) TAKEN TOGETHER are NOT sufficient to answer the question, meaning that
further information would be needed to answer the question.

(C) Statement 1 tells us about Jim's wool sweaters, but we don't know if he has any other types of
sweaters. Eliminate A and D (A and D require that A be able to answer the question). Statement 2 gives
you only a percentage, which is useless in the absence of a real number, eliminate B.
When you combine the statements, you now have a real number of wool sweaters (12) which represents
the other 40% of his total number of sweaters, according to statement 2. Combined, the statements are
sufficient. The answer is C.

Question 11.

If a bottle cap factory makes 7,200 bottle caps per hour, how many does it make per second?

A. 2
B. 120
C. 200
D. 1200
E. 432000

(A) There are 60 minutes in an hour. To figure out how many caps were made in 1 hour, divide 7,200
bottle caps per hour by 60. (7200/60=120 caps per minute).
Since there are 60 seconds in every minute, it makes 120/60 = 2 caps per second. The answer is (A).

Question 12.

Find x + 2y.
(1) x + y = 4
(2) 4x + 8y = 12

A. Statement (1) BY ITSELF is sufficient to answer the question, but statement (2) by itself is not.
B. Statement (2) BY ITSELF is sufficient to answer the question, but statement (1) by itself is not.
C. Statements (1) and (2) TAKEN TOGETHER are sufficient to answer the question, even though
NEITHER statement BY ITSELF is sufficient.
D. Either statement BY ITSELF is sufficient to answer the question.
E. Statements (1) and (2) TAKEN TOGETHER are NOT sufficient to answer the question, meaning that
further information would be needed to answer the question.

(B) You either need to find the values of x and y separately or find the value of the expression x + 2y.
Statement 1 gives you neither, but statement (2) provides an equation (4x + 8y = 12) that, when reduced
(4x + 8y = 12: divide sides by 4), becomes x + 2y = 3, which means that statement 2 answers the question.
Since statement 2 is sufficient and statement 1 is not, the answer is B. Note: C is a trick answer. Indeed
both statements can answer the question, but since statement 2 can do it alone, that is the answer.
Question 13.

How much interest accrued on a $4000 loan over 3 months at 8% annual simple interest?

A. $48.00
B. $80.00
C. $96.00
D. $240.00
E. $320.00

(B) To calculate simple interest, you multiply the principal by the yearly interest rate by the fraction of the
year. The principal is $4000, the interest rate is 8%, and 3 months is ¼ of a year.
Therefore, multiply $4000 × 8/100 × ¼ = 80. The answer is $80, (B).
Question 14.

If 1/3 + 1/2 + 1/x = 4, then x =

A. 18/5
B. 19/6
C. 24/11
D. 6/19
E. 5/18

(D) To solve the equation 1/3 + 1/2 + 1/x = 4, we first need a common denominator to get rid of all of the
fractions. If we multiple both sides of the equation by 6x, we can get rid of all of the fractions.
Multiplying 1/3 + 1/2 + 1/x = 4 by 6x, we get 6x/3 + 6x/2 + 6x/x = 4(6x).
Reducing this, we get 2x + 3x + 6 = 24x
5x + 6 = 24x
6 = 19x, 6/19 = x.

Question 15.

A ranch has horses and ponies. Exactly 5/6 of the ponies have horseshoes. Half of the ponies with
horseshoes are Icelandic ponies. What is the minimum number of ponies on the ranch?

A. 12
B. 15
C. 17
D. 48
E. 54

(A) Time and time again on these tests, you are given very simple questions that look highly intimidating.
Remember that your best asset going into multiple-choice questions is that the answer must be one of the
5 choices. Simply take the answer choices and plug them into the question. Since you are asked for the
minimum number of ponies, you should start with the smallest numbers in the answer choices.
If you have 12 ponies, then 5/6, or 10 ponies have horseshoes. Half of these are Icelandic, making 5
Icelandic ponies. Since these are possible numbers (i.e.- you can't have fractional ponies), 12 is a possible
number of ponies to have, and A is the correct choice. NOTE: all of the other answers are larger, and we
are looking for the minimum, so they may be ruled out. Alternatively, you might note that 5/6 × 1/2 = 5/12
of the ponies are Icelandic. Since the number of ponies must be an integer, the answer is simply the
denominator.

Question 16.

If x + 9 is an even integer, the sum of the next two even integers is:

A. x + 11
B. 2x + 22
C. 2x + 24
D. 3x + 39
E. 24x + 143

(C) The space between two even integers is ALWAYS 2. Accordingly, the next two integers can be
represented by (x + 11) and (x + 13) respectively, which when summed is equal to (2x + 24).

Question 17.

If n is an even integer, which of the following must be an odd integer?

A. 3n - 2
B. 3(n + 1)
C. n - 2
D. n/3
E. n/2

(B) Any time you have variables in the answer choices, you should consider plugging numbers into the
answer choices. For example, let’s make n equal to 2 and then substitute this value into the answer
choices. Try choice B. If n is 2, then 3(n + 1) = 9. Since our target is an odd integer, this answer choice
works. Try a few more numbers to double check. If you insert numbers into all the answer choices, you'll
see that only B always gives an odd number. For example, if you try choice E, (n/2) and substitute 12 for
n, you will get 6, an even number. All of the answer choices except B may be similarly ruled out. Note:
(E) is not correct. Although 2/2 =1 (with n = 2). The question requires that n ALWAYS be odd, regardless
of what even integer it is.

Question 18.

A colony of 15,000 ants increased by 50% after being fed. After 4 weeks without food, the number of ants
dropped by 60%. What percentage decline did the original population undergo?

A. 110%
B. 55%
C. 50%
D. 40%
E. 10%

(D) To solve this question, first we calculate the increase of 50%, then we calculate the decrease of 60%
from the increased amount. Finally, we determine the percentage decline from the original amount.
The colony increases from 15,000 to 15,000 × 1.5 = 22,500. (Increasing something by 50% is the same as
multiplying by 1.5).
After a 60% decline, the colony drops to 9,000 (22,500 × .4 or 22,500 - (.6 × 22,500)). Decreasing
something by 60% is the same as multiplying by .4 (1 - .6 = .4).
Take the original and subtract the later amount: 15,000 – 9,000 = 6,000 decline.
Since 6,000 represents 40% of 15,000 (divide 6,000 by 15,000 to get 40%), the decline is 40%.

Question 20.

The ratio of Marketing to Accounting to Finance majors at Metropolis Business School is 7 : 8 : 10,
respectively. If there are 500 students in the school, how many of them are Finance majors?

A. 140
B. 160
C. 200
D. 250
E. 320

(C) To calculate what the ratio 7 : 8 : 10 means, add 7 + 8 + 10 to get 25. Since Marketing to Accounting
to Finance is 7 : 8 : 10 (where Finance is 10), the ratio tells you that for every 25 people, 10 are Finance
majors. So for every 250 people, 100 are Finance majors, which also means that out of every 500 people,
200 are Finance majors (multiply 500 by 10/25).

Question 21.

Jack has a maximum of $4,000 to spend on a new computer. What is the total cost of the new computer if
the terms of his installment plan are that he must pay $1,000 up front and the remainder of the cost in
equal payments?
(1) The installments are to be $49 per month.
(2) Jack pays 36 installments.

A. Statement (1) BY ITSELF is sufficient to answer the question, but statement (2) by itself is not.
B. Statement (2) BY ITSELF is sufficient to answer the question, but statement (1) by itself is not.
C. Statements (1) and (2) TAKEN TOGETHER are sufficient to answer the question, even though
NEITHER statement BY ITSELF is sufficient.
D. Either statement BY ITSELF is sufficient to answer the question.
E. Statements (1) and (2) TAKEN TOGETHER are NOT sufficient to answer the question, meaning that
further information would be needed to answer the question.
(C) To determine the total cost on the installment plan, you must know either a combination of the total
cost and number of installments or a combination of the number of installments and the amount of each
installment.
Statement 1 tells you how much each installment is, but not how many there are; eliminate choices A and
D (choices A and D assume that A alone is sufficient).
Statement 2 tells you how many installments there are, but not how much each installment is; eliminate B.
Combined, the statements tell you the two things you need (amount of each installment and the number of
payments). Since the question requires two statements, the answer is C.
Email any questions or comments to 24hourtutor@800score.com

Question 22.

A computer manufacturer sold a PC to a retailer for 40% more than it cost to produce. A customer bought
the computer for 25% more than the retailer paid for the computer. The price the customer paid was what
percent greater than the cost of producing the computer?

A. 65%
B. 70%
C. 75%
D. 85%
E. 95%

(C) If the computer cost $100 to make, then the retailer bought it for $140 (40% more than $100). The
customer would then buy it at 25% more than $140, or $175 (to calculate a 25% increase, multiply $140
by 1.25.
Subtract $100 from $175 and the answer, $75, is the percent increase: 75%.
Or, take the original price as 1, multiply by 1.4 for adding 40%, and then add 25% to 1.4 (multiply by
1.25). The result is 1.75, thus 75% higher than the original cost.

Question 23.

What is the perimeter of rectangle Z?


(1) The width of Z is 8.
(2) The length of Z is twice the width of Z.

A. Statement (1) BY ITSELF is sufficient to answer the question, but statement (2) by itself is not.
B. Statement (2) BY ITSELF is sufficient to answer the question, but statement (1) by itself is not.
C. Statements (1) and (2) TAKEN TOGETHER are sufficient to answer the question, even though
NEITHER statement BY ITSELF is sufficient.
D. Either statement BY ITSELF is sufficient to answer the question.
E. Statements (1) and (2) TAKEN TOGETHER are NOT sufficient to answer the question, meaning that
further information would be needed to answer the question.

(C) To know the perimeter of a rectangle, you need the measurements of the length and width.
Statement 1 gives the width, but not the length, so it can't be sufficient. Eliminate AD(choices A and D
assume that A can sufficiently answer the question).
Statement 2 gives you the relationship between the length and width, but no actual numbers, so it is not
sufficient either. Eliminate B. When the two statements are combined, you now have enough information
to solve for the length, and statement 1 gives you width. The answer must be C.

Question 24.

A student planned to complete a college term paper on Wednesday, September 15. The deadline was then
postponed for 48 days. On which day of the week was the task completed?

A. Monday
B. Tuesday
C. Wednesday
D. Thursday
E. Friday

(B) 48 days late is one day shy of exactly 7 weeks (7 weeks times 7 days/week = 49 days). If the job were
finished in 49 days, then it would have been completed on the same day, Wednesday. But since 48 is one
day less than 7 weeks, it was completed one day earlier than Wednesday: Tuesday.

Question 25.

If the angles of a triangle are in the ratio 4 : 3 : 2, then the smallest angle in the triangle must be:

A. 30º
B. 40º
C. 70º
D. 75º
E. 80º

(B) A triangle has 180 degrees. Therefore, our 3 angles sum to 180º. If the 3 angles are in a 4 : 3 : 2 ratio,
then we can create the equation 4s + 3s + 2s = 180º. The smallest angle would then be equal to 2s.
4s + 3s + 2s = 180º
9s = 180
If s = 20, then 2s (the smallest angle) equals 40º.

Question 26.

On a map, 1 inch represents 35 miles. How many inches would be necessary to represent a distance of 245
miles?

A. 5
B. 7
C. 9
D. 210
E. 280

(B). This is a fairly simple proportion problem provided that you keep the relationships clear. If 1 inch is
equal to 35 miles, then X inches is equal to 245 miles can be written as (1/35)= (X/245). When you solve
for X the answer is 7. You could also have easily back solved on this equation.

Question 27.

On two of his tests, Harry scored 30 out of a maximum of 50 and on the third he scored 40 out of a
maximum of 100. What is his average percentage score on the 3 tests?

A. 33.33%
B. 40%
C. 50%
D. 53.3%
E. 66.66%

(D) Harry's percentage scores for the 3 tests are 60%, 60%, and 40% respectively. Thus, the average
percentage is (60+60+40)/3 = 160/3 = 53.33. A common mistake here is to simply take the average of
60% and 40% and incorrectly reach the solution 50%. Since there are two tests in which Harry scores
60%, we take a weighted average, wherein we add 60+60+40 and divide by 3.

Question 28.

Of 70 players on a baseball team, 40 are right-handed. Approximately what percent are not right-handed?

A. 20%
B. 35%
C. 43%
D. 47%
E. 55%

(C) If 40 out of 70 are right-handed, then 30 out of 70 are left-handed. To translate 30 out of 70 into a
percentage, divide 30 by 70. This is .428. To translate this into a percentage, multiply by 100%, to get
42.8%. The closest answer is C, 43%.

Question 29.

A salesman’s income was divided between commission and regular salary. His income, therefore, varied
from week to week. His weekly totals over a 5-week period were $406.20, $413.50, $420.00, $425.00 and
$395.30. What was his average weekly income over the 5 week period?

A. $400.40
B. $408.90
C. $410.40
D. $412.00
E. $2060.00

(D) Add up all the weekly wages: $406.20 + 413.50 + 420.00 + 425.00 + 395.30 = $2060.00. Divide
$2060.00 by 5 to get the average weekly wage, $412.00.

Question 30.

Jennifer, Becky and Susan make contributions to the fundraiser in the ratio of 2:3:5. If their total
contributions are $100,000, how much more money does Susan contribute than Jennifer?

A. $10,000
B. $20,000
C. $30,000
D. $50,000
E. $70,000

(C) Jennifer contributes 2x, Becky 3x and Susan 5x. The total contributed = 100,000 which = 2x + 3x +
5x. Therefore 10x = 100,000 and x = 10,000. The difference between what Susan contributed and what
Jennifer contributed = 5x - 2x which = 3x. Since x = 10,000 we know that 3x = $30,000.

Question 31.

The average (arithmetic mean) of 6 numbers is 24. If the sum of 4 of these numbers is 106, what is the
average of the other two numbers?

A. 12
B. 19
C. 48
D. 72
E. 96

(B) This is an average problem, so use the average formula. If the average of 6 numbers is 24, we can
solve for their sum: 6 × 24 = 144. If 4 of these numbers total 106, then by subtracting 106 from 144, we
get the sum of the other two numbers, 38. To find the average of these two numbers, we divide their sum
by their number: 38/2 = 19.

Question 32.

What is the value of x?


(1) x/y = 4
(2) x - y = 12

A. Statement (1) BY ITSELF is sufficient to answer the question, but statement (2) by itself is not.
B. Statement (2) BY ITSELF is sufficient to answer the question, but statement (1) by itself is not.
C. Statements (1) and (2) TAKEN TOGETHER are sufficient to answer the question, even though
NEITHER statement BY ITSELF is sufficient.
D. Either statement BY ITSELF is sufficient to answer the question.
E. Statements (1) and (2) TAKEN TOGETHER are NOT sufficient to answer the question, meaning that
further information would be needed to answer the question.

(C) Statement 1 (x/y = 4) tells us that x = 4y (by multiplying both sides by y), but that is insufficient
because we need a single value for x.
Statement 2 (x - y = 12) doesn't solve for x either. But if you add the two statements together, the
statement can be solved (C).
Here are the calculations, which are unnecessary because you can always calculate a value from two linear
equations (linear equations are equations without exponents/roots).
Since we know x = 4y, plug in 4y for x in statement 2:
4y - y = 12, 3y = 12, y = 4
Therefore x = 16.

Question 33.

Is the line at the bank teller getting longer or shorter?


(1) People are getting into the line at the rate of 3 people per minute.
(2) The bank teller processes customers at the rate of 4 people every 2 minutes.

A. Statement (1) BY ITSELF is sufficient to answer the question, but statement (2) by itself is not.
B. Statement (2) BY ITSELF is sufficient to answer the question, but statement (1) by itself is not.
C. Statements (1) and (2) TAKEN TOGETHER are sufficient to answer the question, even though
NEITHER statement BY ITSELF is sufficient.
D. Either statement BY ITSELF is sufficient to answer the question.
E. Statements (1) and (2) TAKEN TOGETHER are NOT sufficient to answer the question, meaning that
further information would be needed to answer the question.

(C) Statement 1 tells us the rate at which people are getting into the line, but we need to know at what rate
the customers are being processed. Statement 2 gives us this information. Therefore, the two statements
together are sufficient to see that the net gain in people on the line is 1 per minute; the line is getting
longer.

Question 34.

A taxi company charges $1.75 for the first quarter mile and fifteen cents for each additional quarter mile.
What is the maximum distance you can travel with $4.90?

A. 4 miles
B. 5 1/4 miles
C. 5 1/2 miles
D. 5 3/4 miles
E. 6 1/4 miles
(C) If you start out with $4.90 and have to spend $1.75 for the first quarter mile, you then have $3.15 left
to spend on ¼ mile intervals. Divide $3.15 by $.15 (the cost per quarter mile) to get 21 quarter miles. The
21 quarter miles plus the initial quarter mile = 22 quarter miles. 22 quarter miles = 22 × ¼, or 5.5 miles.

Question 35.

If Henry Ford's Red River factory turns out 10 Model T's in 18 minutes, approximately how many could it
make in one hour?

A. 27
B. 28
C. 30
D. 33
E. 180

(D) If you're stuck for a method, estimate. 18 minutes is a little less than 20 minutes (which is a third of an
hour), so there is a little more than 3 18-minute periods in one hour. In one hour, then, the factory can
produce a little more than 3 times the work that it could do in 18 minutes (which is 10 cars), so your
estimate should be that in one hour the factory can make a little more than 30 cars. The answer must be D.

Question 36.

What is the perimeter of quadrilateral Q?


(1) Q is a rectangle.
(2) The area of Q is 36.

A. Statement (1) BY ITSELF is sufficient to answer the question, but statement (2) by itself is not.
B. Statement (2) BY ITSELF is sufficient to answer the question, but statement (1) by itself is not.
C. Statements (1) and (2) TAKEN TOGETHER are sufficient to answer the question, even though
NEITHER statement BY ITSELF is sufficient.
D. Either statement BY ITSELF is sufficient to answer the question.
E. Statements (1) and (2) TAKEN TOGETHER are NOT sufficient to answer the question, meaning that
further information would be needed to answer the question.

(E) To find the perimeter of a quadrilateral, you must know the lengths of each of the 4 sides.
Statement 1 tells you that opposite sides will be equal, but doesn't tell you anything about the actual
lengths, and so is insufficient. Eliminate choices A and D. Statement 2 tells you the area, which would be
sufficient if Q were a square, but you don't know what type of quadrilateral it actually is.
Statement 2 is not sufficient, so eliminate choice B. Combined, the two statements still don't tell you
anything about the actual lengths of the sides of the rectangle. They could be 4 and 9 or 2 and 18.
Eliminate choice C. Since the statements are not sufficient, the answer must be E.
Question 37.

What is z if 0.303z = 2,727?

A. 9,000
B. 900
C. 90
D. 9
E. 0.9

(A) Multiply both sides by 1000 to eliminate the decimal points:


303z = 2,727,000 divide both sides by 303.
z = 9,000
The trick to this question is that you can quickly look at the figures (and the answer choices) and see that
the answer must involve 9 in it. So the only thing this question asks you to figure out is the number of
zeros with the 9.

You might also like